LSAT and Law School Admissions Forum

Get expert LSAT preparation and law school admissions advice from PowerScore Test Preparation.

 mhsk
  • Posts: 7
  • Joined: Oct 28, 2017
|
#41520
Hi,

I have a general question regarding specific words in Answer Choice A). I was extremely confused with the words "inadequate argument," especially, "inadequate." Under which circumstances can the words "inadequate argument" used to describe a flaw in the argument? If the argument does not logically flow from the premise(s) to the conclusion, can I say that "inadequate argument" has been given? Essentially, can I equate "inadequate argument" with "insufficient/incorrect premise"??

Thank you!

-Rachel
 Luke Haqq
PowerScore Staff
  • PowerScore Staff
  • Posts: 712
  • Joined: Apr 26, 2012
|
#42026
Hi Rachel!

Your view of the meaning of "inadequate argument" seems right to me. "If the argument does not logically flow from the premise(s) to the conclusion," as you write, then yes that would be an inadequate argument. Similarly, if you concluded that there was an "incorrect/insufficient premise," that could be a flaw that prevents the conclusion from following logically from the premises--it could be grounds for finding the flaw of an "inadequate argument."

It's possible that there might be an extra premise--e.g., one that isn't necessary for the conclusion to follow from the rest of the premises. In that case, it's possible that an incorrect/insufficient premise might not establish an inadequate argument. Most of the time, however, questions don't include an extra premise like that. Usually if you see an insufficient premise, that means you'll also have an inadequate argument overall.

For question 14 specifically, the newspaper subscriber concludes "clearly this conclusion is false," referring to Arnot's argument "that by making certain fundamental changes in government we would virtually eliminate our most vexing social ills." The newspaper subscriber might be right that Arnot's argument "depends on the dubious assumption that government can be trusted to act in the interest of the public," but that doesn't establish that Arnot's conclusion is "clearly...false." All it shows is that there would be more premises necessary to make Arnot's argument air-tight. Answer (A)-- "it repudiates a claim merely on the grounds that an inadequate argument has been given for it"--reflects this; it "repudiates" Arnot's claim as being clearly false, on the grounds that this claim needs more argumentation to be plausible. The fact that there's more argumentation needed, though, doesn't establish that the conclusion is false. That's why the newspaper subscriber's reasoning is flawed in the way indicated in (A).

Hope that helps!
 erust2
  • Posts: 18
  • Joined: May 19, 2018
|
#45910
I am having a hard time with this type of answer. Question 11 and question 14 both have an answer choice that indicates an “inadequate argument”. Is there anything in the stimulus of questions 11 and 14 to know that one is directed toward an inadequate argument and the other is not?

*edit*
It looks like question 11 is attacking the person and not any argument. In fact, it doesn’t look like there is an argument in 11. Is that correct? So, an answer for 11 could be one that details attacking the person and not the conclusion. Is that also correct?
 Malila Robinson
PowerScore Staff
  • PowerScore Staff
  • Posts: 296
  • Joined: Feb 01, 2018
|
#45925
Hi erust2,
In #11, the arguments is that the proposal is dishonest. And as you stated the only reason that is given attacks the person rather than the argument, and as such the argument ignores the fact that the proposal would apply to things that happen in the future. Which leads to Answer E. Answer choice B deals with the inadequate argument and in this case the problem was not that the politician's argument was inadequate, or unsupported, the problem was that the author was essentially ignoring focus of the politician's argument (which was to change future behavior) to instead focus on the politician's past behavior as a reason for doubting the worth of the politician's argument.
In #14, (as explained above) the author's conclusion is too strong for the evidence that has been put forth against Arnot's argument, by the author. You would need to add more evidence than what has been provided to prove that "clearly this conclusion is false". The author has shown that there may be a reason to doubt Arnot's conclusion, but the evidence is inadequate to show that the conclusion is 'clearly false'.
Hope that helps,
-Malila
 ieric01
  • Posts: 34
  • Joined: Dec 09, 2019
|
#73794
Hello,

I'm a bit mixed up with identifying Arnot's argument.

From what I understand the conclusion is the 'claim.' And the premise is the 'reasons' given to support the claim.

I thought this sentence "Arnot’s editorial argues that by making certain fundamental changes in government we would virtually eliminate our most vexing social ills” contained both the premise and conclusion.

The premise being “making certain fundamental changes in government.”

And the conclusion “eliminating our most vexing social ills.”

But looking through the forum, I read it's Arnot's conclusion.

What am I getting wrong?
User avatar
 KelseyWoods
PowerScore Staff
  • PowerScore Staff
  • Posts: 1079
  • Joined: Jun 26, 2013
|
#73822
Hi ieric01!

You are correct that the conclusion is the claim the author is trying to support and the premises are the reasons the author gives you to believe that claim. But look a little closer at how you have identified the parts of Arnot's argument here.

If I say to you: "We would eliminate our most vexing social ills."

And you ask, "why?".

And I respond: "Making certain fundamental changes in government."

Would you be satisfied with that response? Does "making certain fundamental changes in government" give you a reason to believe that "we would eliminate our most vexing social ills"?

Premises should answer the question "why should I believe this conclusion?". In this case, what you have identified as the premise does not answer the question why we should believe what you have identified as the conclusion.

The conclusion can also be thought of as the point the author is arguing for. So, in this case, when the newspaper subscriber says "Arnot's editorial argues..." they are really introducing what Arnot's main point (or conclusion) is. We don't actually get the premise for Arnot's argument--which is definitely confusing! But it's because Arnot's argument is being introduced to us by the newspaper subscriber. So we just rely on what the newspaper subscriber tells us: this is the argument Arnot is making, and his conclusion is false. The newspaper subscriber never tells us what evidence Arnot provided in support of his conclusion.

Again, it gets confusing because the term "argument" can be used to refer to both the full structured argument involving premises and a conclusion as well as the main point (or conclusion) being "argued" for. But when you're identifying premises and conclusions, always make sure that your premises answer the question: "why should I believe this conclusion?"

Hope this helps!

Best,
Kelsey
 ieric01
  • Posts: 34
  • Joined: Dec 09, 2019
|
#73834
Kelsey, that was extremely helpful and clear, thank you!

Now that I understand the argument, I put the argument's flaw in my own words...

Love to get your input. Thanks!

===

The Newspaper Subscriber (NS) doesn’t agree with Arnot. He doesn’t believe making fundamental changes in government will solve society’s social problems.

Why?

Because for that to happen, we have to trust the government will act in the people’s best interest.

And NS doesn’t trust the government. If he did, he’d agree with Arnot.

What’s the flaw?

For one, NS doesn’t back up his argument with evidence showing why fundamental changes in government will NOT solve society’s social problems. For us to have been sold on his argument, he should've eliminated all possibilities of Arnot’s argument being true.

For another, we can’t say Arnot's conclusion IS false because it relies on an assumption - whether silly or not. The only thing it CAN do is weaken Arnot’s argument, NOT prove it’s false. Arnot would then have to look into this assumption and see if it affects his argument, before we can fully accept his claim.

The point is…

A lack of evidence or the absence of evidence as they call it, doesn’t prove a claim itself is false. At best, it can weaken it or cast doubt.

If you’re still confused about the flaw, like I was, maybe this could help:

First, let’s restate the flaw.

The flaw was that you can’t prove something is false because of sketchy or missing evidence. For example, NS believes the government is sketchy. He doesn’t trust them to act in the people's best interest. Now whether he’s right or wrong, it doesn’t automatically make Arnot’s argument false. Even if he thinks Arnot is silly for trusting the government, Arnot's argument could still be true.

As of now, we don’t know yet. Arnot, himself, didn't even provide evidence proving why fundamental changes in the gov’t will solve society’s social problems. Maybe he is right, maybe he is wrong. The lesson here is we have no way of knowing yet. And even if NS raised a valid objection, until it’s been tested, we can’t assume Arnot’s claim is false.
 Claire Horan
PowerScore Staff
  • PowerScore Staff
  • Posts: 408
  • Joined: Apr 18, 2016
|
#73841
Hi ieric01,

Good job thinking through this one. I think you are still getting a bit tripped up in the wording, but, rather than beat this question to death, I'll try to give another example with the same structure that employs the same flaw in the reasoning.

Candy Seller: The father's editorial arguing that eating candy causes children to be hyper depends entirely on data from a poorly designed experiment. Therefore, he must be wrong that candy causes hyperactivity in children.

This structure is a bit tricky, as Kelsey noted above, because the Candy Seller's argument is about the father's argument (2 levels of argument!).

Here's the Candy Seller's argument:
Premise 1: The Editorialist (the father) has only one argument in support of his editorial.
Premise 2: The evidence for that argument is data from an untrustworthy study.
(logical gap)
Conclusion: Candy doesn't cause hyperactivity.

What fills the logical gap?

Well, if the Seller's premises are true, the proper conclusion is: The Editorialist's argument is unsound.
Instead, she decides jumps all the way to the conclusion that his conclusion about candy has been proven wrong.

The Seller's flawed logic, phrased more abstractly, is that she assumes that: because the Editorialist father's argument is suspect, the father's conclusion itself must be false.

What I think is still getting you tripped up in #14 is that you are still focusing on whether the Newspaper Subscriber has shown Arnot's argument is unsound. But, given the premises, the Subscriber has shown that Arnot's argument is "inadequate." That's not the problem. The problem is the LOGICAL JUMP from there to a "repudiation" (rejection) of the conclusion that "by making certain fundamental changes in government, we would virtually eliminate our most vexing social ills." If you read answer choice again now, I think it will make more sense to you.

One more fun example:
"You are wrong: Unicorns aren't real. You only believe in them because you thought you saw one, and that was just a horse with something glued to its head."
The speaker is using flawed reasoning by thinking that successfully discounting the listener's evidence of unicorns definitively proves that the unicorn-believer's conclusion (that unicorns exist) is wrong.

I hope these examples helped somewhat! :-D
 ieric01
  • Posts: 34
  • Joined: Dec 09, 2019
|
#73864
Claire, the candy seller example was perfect! Along with all the other gems you dropped, that I was glad to pick up. Thanks!
User avatar
 cgs174
  • Posts: 11
  • Joined: Jan 01, 2022
|
#94136
Hi!! I missed this question on my first take but got it correct on the blind review. In my read through I was surprised by how little information was given about Arnot's argument but my pre-phrase was that even an incorrect assumption, much less a dubious one, does not make a conclusion false. I first went to C) but recognized it was almost the logical inverse of my pre-phrase. I then was between A) and D). I felt that A) was the simple choice for my initial pre-phrase of the flaw but I also noticed a second flaw in the argument which I thought D) responded to well. This second flaw is that the conclusion Arnot makes is simply that by making fundamental changes vexing social ills would be eliminated. The Newspaper subscriber appears to be attacking how likely that is to occur. He seems to be arguing that the changes in government won't be made, not that if they were made it still wouldn't eliminate our most vexing social ills. This move seems to represent Arnot's argument.

On my blind review I choose A) as the more obvious flaw but also because I thought maybe I didn't have enough evidence of Arnot's argument to see how the assumption fits into Arnot's argument. Is this why D) can be ruled out??

Thanks!!!

Get the most out of your LSAT Prep Plus subscription.

Analyze and track your performance with our Testing and Analytics Package.